Exam Details

  • Exam Code
    :USMLE-STEP-3
  • Exam Name
    :United States Medical Licensing Step 3
  • Certification
    :USMLE Certifications
  • Vendor
    :USMLE
  • Total Questions
    :804 Q&As
  • Last Updated
    :Apr 12, 2025

USMLE USMLE Certifications USMLE-STEP-3 Questions & Answers

  • Question 781:

    A 64-year-old male with a history of hypertension and tobacco abuse presents for follow-up after a routine physical during which he was found to have 45 red blood cells (RBCs) per high-power field (HPF) on a screening urinalysis. The urinalysis was negative for leukocytes, nitrites, epithelial cells, and ketones. The patient denies any complaints and the review of systems is essentially negative.

    What would be your initial approach in the workup of this patient with asymptomatic microscopic hematuria?

    A. check PSA and urine culture

    B. CT scan with and without contrast of the abdomen and pelvis

    C. intravenous pyelography (IVP)

    D. observation and reassurance as patient is asymptomatic

    E. repeat urinalysis

  • Question 782:

    A54-year-old Asian female with no significant medical history presents with frontal headache, eye pain, nausea, and vomiting. Her abdominal examination shows mild diffuse tenderness but no rebound or guarding. Her mucous membranes are dry. Her vision is blurry in both eyes, her eyes are injected but her extraocular muscles are intact. Her pupils are mid-dilated and fixed

    Which of the following is the most likely diagnosis?

    A. diabetic ketoacidosis (DKA)

    B. appendicitis

    C. angle closure glaucoma

    D. perforated colon due to inflammatory bowel disease (IBD)

    E. cerebellar malignancy

  • Question 783:

    A 42-year-old man presents to your clinic with a 1-week history of pain and inflammation involving his right first metatarsophalangeal (MTP) joint. He describes the pain as sudden in onset and worse at night. He denies experiencing any fever or traumatic injury to the joint and states that he has never had this type of pain before. He denies any chronic medical conditions, any prior surgery, and any current medication use. Besides an erythematous and exquisitely tender right first MTP joint, the remainder of his physical examination is unremarkable.

    Which of the following is true of the patient's condition?

    A. It commonly presents in premenopausal women.

    B. It commonly presents as a monoarticular arthritis.

    C. Episodes of pain and inflammation become more frequent but resolve more quickly as the disease progresses.

    D. The presence of tophi is a common early finding.

    E. A blood test is the diagnostic gold standard.

  • Question 784:

    A28-year-old male, well known to your clinic, presents for management of swelling, pain, and tenderness that has developed in his left ankle and right knee. It has persisted for 1 month. Your patient reports that he developed severe diarrhea after a picnic 1 month prior to the onset of his arthritis. During the interval between the diarrhea and onset of arthritis, he developed a "pink eye" that lasted for 4 days. He denies any symptoms of back pain or stiffness. You remember that he was treated with ceftriaxone and doxycycline for gonorrhea 2 years ago, which he acquired from sexual activity with multiple partners. Since that time, he has been in a monogamous relationship with his wife and has not had any genitourinary symptoms. He promises that he has been faithful to his wife and has not engaged in unprotected sexual activity outside his marriage. His physical examination is notable for a swollen left ankle, swollen right knee, and the absence of penile discharge or any skin lesions. The patient's symptoms do not respond to your initial therapeutic management. You suspect that his condition is refractory to treatment. Which of the following should you consider at this time?

    A. He may have human immunodeficiency virus (HIV) infection and should be tested.

    B. His condition will require high doses of prednisone (60 mg daily) for adequate control.

    C. His joints are obviously not infected and should be directly injected with corticosteroids.

    D. He must have a disseminated bacterial infection that will require IV antibiotics.

    E. He is resistant to indomethacin, so the dose should be doubled to 400 mg daily.

  • Question 785:

    A28-year-old male, well known to your clinic, presents for management of swelling, pain, and tenderness that has developed in his left ankle and right knee. It has persisted for 1 month. Your patient reports that he developed severe diarrhea after a picnic 1 month prior to the onset of his arthritis. During the interval between the diarrhea and onset of arthritis, he developed a "pink eye" that lasted for 4 days. He denies any symptoms of back pain or stiffness. You remember that he was treated with ceftriaxone and doxycycline for gonorrhea 2 years ago, which he acquired from sexual activity with multiple partners. Since that time, he has been in a monogamous relationship with his wife and has not had any genitourinary symptoms. He promises that he has been faithful to his wife and has not engaged in unprotected sexual activity outside his marriage. His physical examination is notable for a swollen left ankle, swollen right knee, and the absence of penile discharge or any skin lesions. What would be the appropriate management for this patient's arthritis?

    A. Screen him for the suspected disease with HLA-B27 testing.

    B. Treat with daily indomethacin (150200 mg daily).

    C. Start him on empiric antibiotics.

    D. Start treatment with prednisone 10 mg daily.

    E. Assume that the patient is not being honest and perform the appropriate urogenital testing to confirm gonorrhea.

  • Question 786:

    A28-year-old male, well known to your clinic, presents for management of swelling, pain, and tenderness that has developed in his left ankle and right knee. It has persisted for 1 month. Your patient reports that he developed severe diarrhea after a picnic 1 month prior to the onset of his arthritis. During the interval between the diarrhea and onset of arthritis, he developed a "pink eye" that lasted for 4 days. He denies any symptoms of back pain or stiffness. You remember that he was treated with ceftriaxone and doxycycline for gonorrhea 2 years ago, which he acquired from sexual activity with multiple partners. Since that time, he has been in a monogamous relationship with his wife and has not had any genitourinary symptoms. He promises that he has been faithful to his wife and has not engaged in unprotected sexual activity outside his marriage. His physical examination is notable for a swollen left ankle, swollen right knee, and the absence of penile discharge or any skin lesions. Which of the following is the most likely diagnosis?

    A. pseudogout

    B. gout

    C. reactive arthritis

    D. resistant gonococcal arthritis

    E. ankylosing spondylitis

  • Question 787:

    A 60-year-old woman arrives at your office for a routine physical examination. During the course of her examination she asks you about osteoporosis. She is concerned about her risk for osteoporosis, as her mother suffered from multiple vertebral compression fractures at the age of 60. Your patient reports that she still smokes cigarettes ("although I know they are bad for me") and has one alcoholic beverage a week. She reports having had menopause 5 years ago and experiencing a deep venous thrombosis approximately 20 years ago. She is proud of the fact that she regularly exercises at the local fitness center. She has been taking 1500 mg of calcium with 800 IU of vitamin D every day. You suspect that she is at risk for osteoporosis. After a thorough discussion with your patient, you determine that pharmacologic intervention would be beneficial given the severity of her osteoporosis. Which of the following is most appropriate for your patient?

    A. estrogen replacement therapy

    B. combined HRT with estrogen and progestin

    C. alendronate

    D. calcitonin intranasal spray

    E. raloxifene

  • Question 788:

    A 60-year-old woman arrives at your office for a routine physical examination. During the course of her examination she asks you about osteoporosis. She is concerned about her risk for osteoporosis, as her mother suffered from multiple vertebral compression fractures at the age of 60. Your patient reports that she still smokes cigarettes ("although I know they are bad for me") and has one alcoholic beverage a week. She reports having had menopause 5 years ago and experiencing a deep venous thrombosis approximately 20 years ago. She is proud of the fact that she regularly exercises at the local fitness center. She has been taking 1500 mg of calcium with 800 IU of vitamin D every day. You suspect that she is at risk for osteoporosis. After performing the appropriate imaging study, you determine that your patient has osteoporosis. Of the following choices, which is risk factor most likely contributing to her osteoporosis?

    A. active lifestyle

    B. late menopause

    C. cigarette smoking

    D. frequency of alcohol intake

    E. her intake of calcium and vitamin D

  • Question 789:

    A 60-year-old woman arrives at your office for a routine physical examination. During the course of her examination she asks you about osteoporosis. She is concerned about her risk for osteoporosis, as her mother suffered from multiple vertebral compression fractures at the age of 60. Your patient reports that she still smokes cigarettes ("although I know they are bad for me") and has one alcoholic beverage a week.

    She reports having had menopause 5 years ago and experiencing a deep venous thrombosis

    approximately 20 years ago. She is proud of the fact that she regularly exercises at the local fitness center.

    She has been taking 1500 mg of calcium with 800 IU of vitamin D every day. You suspect that she is at risk

    for osteoporosis.

    Which of the following tests is best to detect and monitor osteoporosis?

    A. plain film radiography

    B. dual photon absorptiometry

    C. single photon absorptiometry

    D. dual-energy x-ray absorptiometry (DEXA)

    E. quantitative CT scan

  • Question 790:

    A 65-year-old man presents to your office for evaluation of abdominal pain. The patient states that he has epigastric pain that radiates to his back. The pain is worse with eating and improves with fasting. The pain has been present for 6 months and is gradually worsening. The patient has lost 15 lbs but feels his oral intake has been adequate. He complains of greasy stools and frequent thirst and urination. Examination reveals a thin male with temporal wasting and oderate abdominal pain with palpation. The patient consumes approximately 1015 beers per day and smokes a pack of cigarettes per day for the past 20 years. The patient's abdominal pain worsens and his weight loss progresses despite therapy, and you suspect that he may have a malignancy. If a malignancy was present, which tumor marker would be most likely to be elevated in this patient?

    A. carcinoembryonic antigen (CEA)

    B. prostate-specific antigen (PSA)

    C. cancer antigen (CA)-125

    D. -Fetoprotein (AFP)

    E. CA-19-9

Tips on How to Prepare for the Exams

Nowadays, the certification exams become more and more important and required by more and more enterprises when applying for a job. But how to prepare for the exam effectively? How to prepare for the exam in a short time with less efforts? How to get a ideal result and how to find the most reliable resources? Here on Vcedump.com, you will find all the answers. Vcedump.com provide not only USMLE exam questions, answers and explanations but also complete assistance on your exam preparation and certification application. If you are confused on your USMLE-STEP-3 exam preparations and USMLE certification application, do not hesitate to visit our Vcedump.com to find your solutions here.